Find all values such that the inequality is true

  • Thread starter Thread starter JOATMON
  • Start date Start date
  • Tags Tags
    Inequality
Click For Summary
The discussion focuses on solving the inequality (n² + 2n + 3) / (2n³ + 5n² + 8n + 3) < 0.025 for natural numbers n. The rational function simplifies to 1/(2n + 1), leading to the conclusion that n must be greater than 19.5, thus n must be at least 20. Participants emphasize the importance of showing detailed steps in the solution process and suggest flipping the inequality for simplification. It is also noted that checking the signs of both sides of the inequality is crucial for ensuring valid solutions. The final conclusion is that the inequality holds true for all natural numbers n greater than or equal to 20.
JOATMON
Messages
2
Reaction score
0
Mod note: Moved from a technical math section, so missing the homework template.
This is for an Intro to Analysis course. It's been a very long time since I've taken a math course, so I do not remember much of anything.

=============
Here is the problem:For the inequality below, find all values n ∈ N such that the inequality is true:

(n2 + 2n +3) / (2n3 + 5n2 + 8n + 3) < 0.025.============================
Here is my attempt at the problem:Looking at the following set

{n ∈ N: (n2 + 2n +3) / (2n3 + 5n2 + 8n + 3) < 0.025}We want to find the lower bound of this set.Suppose A denotes the above set, then we haveA= {n ∈ N: (n2 + 2n +3) / (2n3 + 5n2 + 8n + 3) < 0.025}Since the above rational function can be reduced to 1/(2n+1) we have1/(2n+1) <0.025Where we get n>19.5. Since the lower bounds of the set are 19.5, 19.4, 19.3... And so on, this set is bound below by the natural number n=19. Therefore, the above inequality holds true for all n ∈ N greater than or equal to 20.==========
If you can only just tell me what topics I need to review to answer this correctly, I would appreciate it.
 
Last edited by a moderator:
Physics news on Phys.org
what is n2,n3?
for example if n=1, n2=1 and n3=170000 you can make the inequality true...
\frac{1+2+3}{2 \times 170000 + 5 + 8 +3} = \frac{6}{340016}= 0.00001764622 &lt; 0.025
and so n=1 is keeping the inequality
 
ChrisVer said:
what is n2,n3?

Sorry. I didn't check to see if the format changed once I copied and pasted my problem:
==============================================================

Here is the problem:For the inequality below, find all values n ∈ N such that the inequality is true:

(n2 + 2n +3) / (2n3 + 5n2+ 8n + 3) < 0.025.============================
Here is my attempt at the problem:Looking at the following set

{n ∈ N: (n2+ 2n +3) / (2n3 + 5n2 + 8n + 3) < 0.025}We want to find the lower bound of this set.Suppose A denotes the above set, then we haveA= {n ∈ N: (n2 + 2n +3) / (2n3 + 5n2 + 8n + 3) < 0.025}Since the above rational function can be reduced to 1/(2n+1) we have1/(2n+1) <0.025Where we get n>19.5. Since the lower bounds of the set are 19.5, 19.4, 19.3... And so on, this set is bound below by the natural number n=19. Therefore, the above inequality holds true for all n ∈ N greater than or equal to 20. Thus A= {20, 21, 22, 23,...}
 
I don't see the purpose of all those "steps" before reducing the rational function (which you should probably show in more detail).
JOATMON said:
Where we get n>19.5
Okay (showing the steps wouldn't hurt).
JOATMON said:
Since the lower bounds of the set are 19.5, 19.4, 19.3... And so on
What is the relevance of those numbers?

Once you have n>19.5 you can directly conclude that n>=20, and write down the set of solutions.
 
One method that might help with the simplification is to flip the inequality.
##\frac{ n^2 +2n+3}{2n^3 + 5n^2 + 8n+3} < .025 \equiv \frac{2n^3 + 5n^2 + 8n+3}{ n^2 +2n+3}>40##
Which as you pointed out can be written as
##2n+1 > 40.##
And you already have the solution.
 
  • Like
Likes PeroK
Be careful: to do that you have to check that the two sides cannot be negative. This is easy to do here, but it is a necessary step.
 
Question: A clock's minute hand has length 4 and its hour hand has length 3. What is the distance between the tips at the moment when it is increasing most rapidly?(Putnam Exam Question) Answer: Making assumption that both the hands moves at constant angular velocities, the answer is ## \sqrt{7} .## But don't you think this assumption is somewhat doubtful and wrong?

Similar threads

  • · Replies 8 ·
Replies
8
Views
2K
  • · Replies 25 ·
Replies
25
Views
3K
  • · Replies 2 ·
Replies
2
Views
1K
  • · Replies 2 ·
Replies
2
Views
2K
  • · Replies 13 ·
Replies
13
Views
4K
Replies
3
Views
3K
  • · Replies 4 ·
Replies
4
Views
4K
Replies
11
Views
3K
  • · Replies 15 ·
Replies
15
Views
3K
  • · Replies 1 ·
Replies
1
Views
2K